Page 1 of 1

LRB (2017 ed.) Page 138

Posted: Thu Aug 24, 2017 6:40 pm
by alecscook
Hello,

I'm having difficulty understanding why B was the correct answer choice for problem #7 on page 132. I initially thought it was B, but the stimulus doesn't explicitly discuss whether the molecular motion of rhodopsin are proportionate to high or low temperatures. That's what threw me off with this question. I wasn't sure whether they were proportionate to high temperatures or to low temperatures, and answer B assumes that they're more prone to error in hotter temperatures. To me this answer was a reach of information that I could not firmly say was true. What did I miss here? Thanks in advance.

Re: LRB (2017 ed.) Page 138

Posted: Fri Aug 25, 2017 12:40 pm
by Dave Killoran
Hi Alec,

This is a tough problem, and one that I've expanded the explanation to over the years in response to thoughtful student questions like yours. We've also been asked about this a few times here on the forum, so I'm going to start by referring you to two threads that talk about the proportional idea:

lsat/viewtopic.php?t=6831&p=17176

lsat/viewtopic.php?t=6355&p=16016

If you wouldn't mind, please take a look at those and let me know if they help answer your question. If not, I'll come back through and go more deeply into the discussion with you.

Thanks!